Difference between revisions of "2011 AMC 8 Problems/Problem 17"
m |
|||
Line 4: | Line 4: | ||
==Solution== | ==Solution== | ||
+ | The [[prime factorization]] of <math>588</math> is <math>2^2\cdot3\cdot7^2.</math> We can see <math>w=2, x=1,</math> and <math>z=2.</math> Because <math>5^0=1, y=0.</math> | ||
+ | |||
+ | <cmath>2w+3x+5y+7z=4+3+0+14=\boxed{\textbf{(A)}\ 21}</cmath> | ||
==See Also== | ==See Also== | ||
{{AMC8 box|year=2011|num-b=16|num-a=18}} | {{AMC8 box|year=2011|num-b=16|num-a=18}} |
Revision as of 19:30, 25 November 2011
Let , , , and be whole numbers. If , then what does equal?
Solution
The prime factorization of is We can see and Because
See Also
2011 AMC 8 (Problems • Answer Key • Resources) | ||
Preceded by Problem 16 |
Followed by Problem 18 | |
1 • 2 • 3 • 4 • 5 • 6 • 7 • 8 • 9 • 10 • 11 • 12 • 13 • 14 • 15 • 16 • 17 • 18 • 19 • 20 • 21 • 22 • 23 • 24 • 25 | ||
All AJHSME/AMC 8 Problems and Solutions |